Pythagorean Triplets with “Bounds”











up vote
3
down vote

favorite












I am interested in the algebraic/geometric way of finding the pythagorean triplets such that



$$a^2 + b^2 = c^2$$



$$a + b + c = 1000$$



I do the obvious



$$a + b = 1000 - (a^2 + b^2)^{1/2}$$



$$a^2 + b^2 = 1000^2 -2(1000)a - 2(1000)b +2ab + a^2 +b^2$$



$$2a + 2b - frac{2ab}{1000} = 1000$$



$$a + b -frac{ab}{1000} = 500$$



I have no idea what to do at this point. I can't separate the variables, and any geometric solution is beyond my reach. The only other thing I can think of is writing



$$a + b = 500 + frac{ab}{1000} = 1000 - c$$



But this is going backwards










share|cite|improve this question
























  • Only 1 exists right? look at answers.yahoo.com/question/index?qid=20091110033358AAfNaHy and Euclids formula en.wikipedia.org/wiki/Pythagorean_triple
    – Sidd Singal
    Aug 7 '12 at 16:38












  • Do you want to include solutions with negative values like $a=1500$, $b=2000$, and $c=-2500$?
    – user940
    Aug 7 '12 at 17:01















up vote
3
down vote

favorite












I am interested in the algebraic/geometric way of finding the pythagorean triplets such that



$$a^2 + b^2 = c^2$$



$$a + b + c = 1000$$



I do the obvious



$$a + b = 1000 - (a^2 + b^2)^{1/2}$$



$$a^2 + b^2 = 1000^2 -2(1000)a - 2(1000)b +2ab + a^2 +b^2$$



$$2a + 2b - frac{2ab}{1000} = 1000$$



$$a + b -frac{ab}{1000} = 500$$



I have no idea what to do at this point. I can't separate the variables, and any geometric solution is beyond my reach. The only other thing I can think of is writing



$$a + b = 500 + frac{ab}{1000} = 1000 - c$$



But this is going backwards










share|cite|improve this question
























  • Only 1 exists right? look at answers.yahoo.com/question/index?qid=20091110033358AAfNaHy and Euclids formula en.wikipedia.org/wiki/Pythagorean_triple
    – Sidd Singal
    Aug 7 '12 at 16:38












  • Do you want to include solutions with negative values like $a=1500$, $b=2000$, and $c=-2500$?
    – user940
    Aug 7 '12 at 17:01













up vote
3
down vote

favorite









up vote
3
down vote

favorite











I am interested in the algebraic/geometric way of finding the pythagorean triplets such that



$$a^2 + b^2 = c^2$$



$$a + b + c = 1000$$



I do the obvious



$$a + b = 1000 - (a^2 + b^2)^{1/2}$$



$$a^2 + b^2 = 1000^2 -2(1000)a - 2(1000)b +2ab + a^2 +b^2$$



$$2a + 2b - frac{2ab}{1000} = 1000$$



$$a + b -frac{ab}{1000} = 500$$



I have no idea what to do at this point. I can't separate the variables, and any geometric solution is beyond my reach. The only other thing I can think of is writing



$$a + b = 500 + frac{ab}{1000} = 1000 - c$$



But this is going backwards










share|cite|improve this question















I am interested in the algebraic/geometric way of finding the pythagorean triplets such that



$$a^2 + b^2 = c^2$$



$$a + b + c = 1000$$



I do the obvious



$$a + b = 1000 - (a^2 + b^2)^{1/2}$$



$$a^2 + b^2 = 1000^2 -2(1000)a - 2(1000)b +2ab + a^2 +b^2$$



$$2a + 2b - frac{2ab}{1000} = 1000$$



$$a + b -frac{ab}{1000} = 500$$



I have no idea what to do at this point. I can't separate the variables, and any geometric solution is beyond my reach. The only other thing I can think of is writing



$$a + b = 500 + frac{ab}{1000} = 1000 - c$$



But this is going backwards







algebra-precalculus






share|cite|improve this question















share|cite|improve this question













share|cite|improve this question




share|cite|improve this question








edited Aug 7 '12 at 16:41









Micah

29.5k1363104




29.5k1363104










asked Aug 7 '12 at 16:33









Cactus BAMF

5441920




5441920












  • Only 1 exists right? look at answers.yahoo.com/question/index?qid=20091110033358AAfNaHy and Euclids formula en.wikipedia.org/wiki/Pythagorean_triple
    – Sidd Singal
    Aug 7 '12 at 16:38












  • Do you want to include solutions with negative values like $a=1500$, $b=2000$, and $c=-2500$?
    – user940
    Aug 7 '12 at 17:01


















  • Only 1 exists right? look at answers.yahoo.com/question/index?qid=20091110033358AAfNaHy and Euclids formula en.wikipedia.org/wiki/Pythagorean_triple
    – Sidd Singal
    Aug 7 '12 at 16:38












  • Do you want to include solutions with negative values like $a=1500$, $b=2000$, and $c=-2500$?
    – user940
    Aug 7 '12 at 17:01
















Only 1 exists right? look at answers.yahoo.com/question/index?qid=20091110033358AAfNaHy and Euclids formula en.wikipedia.org/wiki/Pythagorean_triple
– Sidd Singal
Aug 7 '12 at 16:38






Only 1 exists right? look at answers.yahoo.com/question/index?qid=20091110033358AAfNaHy and Euclids formula en.wikipedia.org/wiki/Pythagorean_triple
– Sidd Singal
Aug 7 '12 at 16:38














Do you want to include solutions with negative values like $a=1500$, $b=2000$, and $c=-2500$?
– user940
Aug 7 '12 at 17:01




Do you want to include solutions with negative values like $a=1500$, $b=2000$, and $c=-2500$?
– user940
Aug 7 '12 at 17:01










2 Answers
2






active

oldest

votes

















up vote
1
down vote



accepted










The following is copy and pasted directly from Yahoo Answers



All Pythagorean triples are generated by ${m^2+n^2, m^2-n^2, 2mn}$, where $m$ and $n$ are positive integers, and $mgt n$.



You need $a+b+c=1000$, yielding $m(m+n)=500$. So, $m$ and $(m+n)$ are factors of $500$.



$m+ngt m$, so $m(m+n)gt m^2$, so $mlt sqrt{(500)}$,
and
$m+nlt2m$ (since $mgt n$), so $m(m+n)lt2m^2$, so $mgt sqrt{(250)}$.



The prime factorisation of 500 is $2 cdot 2 cdot 5 cdot 5 cdot 5$, so the only factor of $500$ between $15.8$ and $22.4$ is $20$.
Thus, $m=20$, $n=5$, giving the answer required:
$m^2+n^2$ $= 425$;
$m^2-n^2 = 375$;
$2mn = 200$.






share|cite|improve this answer






























    up vote
    1
    down vote













    We know the parametric form of the Pythagorian triplet is $(k(p^2-q^2), 2kpq, k(p^2+q^2))$ where p,r integers.



    So, here $k(p^2-q^2+ 2pq + p^2+q^2)=1000$



    =>$kp(p+q)=500$



    Clearly, k can be any divisor of 500.



    If k=1,



    If p=1, p+q=500=> q = 499,



    if p=2, p+q=250=> q = 248 and so on.



    If k=2,$p(p+q)=250$



    If k=5,$p(p+q)=100$



    If k=10, $p(p+q)=50$ and so on.






    share|cite|improve this answer























    • You don't get all of the non-primitive triples this way (e.g. $9^2 + 12^2 = 15^2$).
      – Qiaochu Yuan
      Aug 7 '12 at 16:43












    • @QiaochuYuan, I've rectified the answer and I think the result will be exhaustive if negative values are permitted.
      – lab bhattacharjee
      Aug 7 '12 at 18:00











    Your Answer





    StackExchange.ifUsing("editor", function () {
    return StackExchange.using("mathjaxEditing", function () {
    StackExchange.MarkdownEditor.creationCallbacks.add(function (editor, postfix) {
    StackExchange.mathjaxEditing.prepareWmdForMathJax(editor, postfix, [["$", "$"], ["\\(","\\)"]]);
    });
    });
    }, "mathjax-editing");

    StackExchange.ready(function() {
    var channelOptions = {
    tags: "".split(" "),
    id: "69"
    };
    initTagRenderer("".split(" "), "".split(" "), channelOptions);

    StackExchange.using("externalEditor", function() {
    // Have to fire editor after snippets, if snippets enabled
    if (StackExchange.settings.snippets.snippetsEnabled) {
    StackExchange.using("snippets", function() {
    createEditor();
    });
    }
    else {
    createEditor();
    }
    });

    function createEditor() {
    StackExchange.prepareEditor({
    heartbeatType: 'answer',
    convertImagesToLinks: true,
    noModals: true,
    showLowRepImageUploadWarning: true,
    reputationToPostImages: 10,
    bindNavPrevention: true,
    postfix: "",
    imageUploader: {
    brandingHtml: "Powered by u003ca class="icon-imgur-white" href="https://imgur.com/"u003eu003c/au003e",
    contentPolicyHtml: "User contributions licensed under u003ca href="https://creativecommons.org/licenses/by-sa/3.0/"u003ecc by-sa 3.0 with attribution requiredu003c/au003e u003ca href="https://stackoverflow.com/legal/content-policy"u003e(content policy)u003c/au003e",
    allowUrls: true
    },
    noCode: true, onDemand: true,
    discardSelector: ".discard-answer"
    ,immediatelyShowMarkdownHelp:true
    });


    }
    });














    draft saved

    draft discarded


















    StackExchange.ready(
    function () {
    StackExchange.openid.initPostLogin('.new-post-login', 'https%3a%2f%2fmath.stackexchange.com%2fquestions%2f179997%2fpythagorean-triplets-with-bounds%23new-answer', 'question_page');
    }
    );

    Post as a guest















    Required, but never shown

























    2 Answers
    2






    active

    oldest

    votes








    2 Answers
    2






    active

    oldest

    votes









    active

    oldest

    votes






    active

    oldest

    votes








    up vote
    1
    down vote



    accepted










    The following is copy and pasted directly from Yahoo Answers



    All Pythagorean triples are generated by ${m^2+n^2, m^2-n^2, 2mn}$, where $m$ and $n$ are positive integers, and $mgt n$.



    You need $a+b+c=1000$, yielding $m(m+n)=500$. So, $m$ and $(m+n)$ are factors of $500$.



    $m+ngt m$, so $m(m+n)gt m^2$, so $mlt sqrt{(500)}$,
    and
    $m+nlt2m$ (since $mgt n$), so $m(m+n)lt2m^2$, so $mgt sqrt{(250)}$.



    The prime factorisation of 500 is $2 cdot 2 cdot 5 cdot 5 cdot 5$, so the only factor of $500$ between $15.8$ and $22.4$ is $20$.
    Thus, $m=20$, $n=5$, giving the answer required:
    $m^2+n^2$ $= 425$;
    $m^2-n^2 = 375$;
    $2mn = 200$.






    share|cite|improve this answer



























      up vote
      1
      down vote



      accepted










      The following is copy and pasted directly from Yahoo Answers



      All Pythagorean triples are generated by ${m^2+n^2, m^2-n^2, 2mn}$, where $m$ and $n$ are positive integers, and $mgt n$.



      You need $a+b+c=1000$, yielding $m(m+n)=500$. So, $m$ and $(m+n)$ are factors of $500$.



      $m+ngt m$, so $m(m+n)gt m^2$, so $mlt sqrt{(500)}$,
      and
      $m+nlt2m$ (since $mgt n$), so $m(m+n)lt2m^2$, so $mgt sqrt{(250)}$.



      The prime factorisation of 500 is $2 cdot 2 cdot 5 cdot 5 cdot 5$, so the only factor of $500$ between $15.8$ and $22.4$ is $20$.
      Thus, $m=20$, $n=5$, giving the answer required:
      $m^2+n^2$ $= 425$;
      $m^2-n^2 = 375$;
      $2mn = 200$.






      share|cite|improve this answer

























        up vote
        1
        down vote



        accepted







        up vote
        1
        down vote



        accepted






        The following is copy and pasted directly from Yahoo Answers



        All Pythagorean triples are generated by ${m^2+n^2, m^2-n^2, 2mn}$, where $m$ and $n$ are positive integers, and $mgt n$.



        You need $a+b+c=1000$, yielding $m(m+n)=500$. So, $m$ and $(m+n)$ are factors of $500$.



        $m+ngt m$, so $m(m+n)gt m^2$, so $mlt sqrt{(500)}$,
        and
        $m+nlt2m$ (since $mgt n$), so $m(m+n)lt2m^2$, so $mgt sqrt{(250)}$.



        The prime factorisation of 500 is $2 cdot 2 cdot 5 cdot 5 cdot 5$, so the only factor of $500$ between $15.8$ and $22.4$ is $20$.
        Thus, $m=20$, $n=5$, giving the answer required:
        $m^2+n^2$ $= 425$;
        $m^2-n^2 = 375$;
        $2mn = 200$.






        share|cite|improve this answer














        The following is copy and pasted directly from Yahoo Answers



        All Pythagorean triples are generated by ${m^2+n^2, m^2-n^2, 2mn}$, where $m$ and $n$ are positive integers, and $mgt n$.



        You need $a+b+c=1000$, yielding $m(m+n)=500$. So, $m$ and $(m+n)$ are factors of $500$.



        $m+ngt m$, so $m(m+n)gt m^2$, so $mlt sqrt{(500)}$,
        and
        $m+nlt2m$ (since $mgt n$), so $m(m+n)lt2m^2$, so $mgt sqrt{(250)}$.



        The prime factorisation of 500 is $2 cdot 2 cdot 5 cdot 5 cdot 5$, so the only factor of $500$ between $15.8$ and $22.4$ is $20$.
        Thus, $m=20$, $n=5$, giving the answer required:
        $m^2+n^2$ $= 425$;
        $m^2-n^2 = 375$;
        $2mn = 200$.







        share|cite|improve this answer














        share|cite|improve this answer



        share|cite|improve this answer








        edited Nov 19 at 20:38









        Ben Brian

        1034




        1034










        answered Aug 7 '12 at 16:47









        Sidd Singal

        2,56231630




        2,56231630






















            up vote
            1
            down vote













            We know the parametric form of the Pythagorian triplet is $(k(p^2-q^2), 2kpq, k(p^2+q^2))$ where p,r integers.



            So, here $k(p^2-q^2+ 2pq + p^2+q^2)=1000$



            =>$kp(p+q)=500$



            Clearly, k can be any divisor of 500.



            If k=1,



            If p=1, p+q=500=> q = 499,



            if p=2, p+q=250=> q = 248 and so on.



            If k=2,$p(p+q)=250$



            If k=5,$p(p+q)=100$



            If k=10, $p(p+q)=50$ and so on.






            share|cite|improve this answer























            • You don't get all of the non-primitive triples this way (e.g. $9^2 + 12^2 = 15^2$).
              – Qiaochu Yuan
              Aug 7 '12 at 16:43












            • @QiaochuYuan, I've rectified the answer and I think the result will be exhaustive if negative values are permitted.
              – lab bhattacharjee
              Aug 7 '12 at 18:00















            up vote
            1
            down vote













            We know the parametric form of the Pythagorian triplet is $(k(p^2-q^2), 2kpq, k(p^2+q^2))$ where p,r integers.



            So, here $k(p^2-q^2+ 2pq + p^2+q^2)=1000$



            =>$kp(p+q)=500$



            Clearly, k can be any divisor of 500.



            If k=1,



            If p=1, p+q=500=> q = 499,



            if p=2, p+q=250=> q = 248 and so on.



            If k=2,$p(p+q)=250$



            If k=5,$p(p+q)=100$



            If k=10, $p(p+q)=50$ and so on.






            share|cite|improve this answer























            • You don't get all of the non-primitive triples this way (e.g. $9^2 + 12^2 = 15^2$).
              – Qiaochu Yuan
              Aug 7 '12 at 16:43












            • @QiaochuYuan, I've rectified the answer and I think the result will be exhaustive if negative values are permitted.
              – lab bhattacharjee
              Aug 7 '12 at 18:00













            up vote
            1
            down vote










            up vote
            1
            down vote









            We know the parametric form of the Pythagorian triplet is $(k(p^2-q^2), 2kpq, k(p^2+q^2))$ where p,r integers.



            So, here $k(p^2-q^2+ 2pq + p^2+q^2)=1000$



            =>$kp(p+q)=500$



            Clearly, k can be any divisor of 500.



            If k=1,



            If p=1, p+q=500=> q = 499,



            if p=2, p+q=250=> q = 248 and so on.



            If k=2,$p(p+q)=250$



            If k=5,$p(p+q)=100$



            If k=10, $p(p+q)=50$ and so on.






            share|cite|improve this answer














            We know the parametric form of the Pythagorian triplet is $(k(p^2-q^2), 2kpq, k(p^2+q^2))$ where p,r integers.



            So, here $k(p^2-q^2+ 2pq + p^2+q^2)=1000$



            =>$kp(p+q)=500$



            Clearly, k can be any divisor of 500.



            If k=1,



            If p=1, p+q=500=> q = 499,



            if p=2, p+q=250=> q = 248 and so on.



            If k=2,$p(p+q)=250$



            If k=5,$p(p+q)=100$



            If k=10, $p(p+q)=50$ and so on.







            share|cite|improve this answer














            share|cite|improve this answer



            share|cite|improve this answer








            edited Aug 7 '12 at 16:53

























            answered Aug 7 '12 at 16:41









            lab bhattacharjee

            222k15155273




            222k15155273












            • You don't get all of the non-primitive triples this way (e.g. $9^2 + 12^2 = 15^2$).
              – Qiaochu Yuan
              Aug 7 '12 at 16:43












            • @QiaochuYuan, I've rectified the answer and I think the result will be exhaustive if negative values are permitted.
              – lab bhattacharjee
              Aug 7 '12 at 18:00


















            • You don't get all of the non-primitive triples this way (e.g. $9^2 + 12^2 = 15^2$).
              – Qiaochu Yuan
              Aug 7 '12 at 16:43












            • @QiaochuYuan, I've rectified the answer and I think the result will be exhaustive if negative values are permitted.
              – lab bhattacharjee
              Aug 7 '12 at 18:00
















            You don't get all of the non-primitive triples this way (e.g. $9^2 + 12^2 = 15^2$).
            – Qiaochu Yuan
            Aug 7 '12 at 16:43






            You don't get all of the non-primitive triples this way (e.g. $9^2 + 12^2 = 15^2$).
            – Qiaochu Yuan
            Aug 7 '12 at 16:43














            @QiaochuYuan, I've rectified the answer and I think the result will be exhaustive if negative values are permitted.
            – lab bhattacharjee
            Aug 7 '12 at 18:00




            @QiaochuYuan, I've rectified the answer and I think the result will be exhaustive if negative values are permitted.
            – lab bhattacharjee
            Aug 7 '12 at 18:00


















            draft saved

            draft discarded




















































            Thanks for contributing an answer to Mathematics Stack Exchange!


            • Please be sure to answer the question. Provide details and share your research!

            But avoid



            • Asking for help, clarification, or responding to other answers.

            • Making statements based on opinion; back them up with references or personal experience.


            Use MathJax to format equations. MathJax reference.


            To learn more, see our tips on writing great answers.





            Some of your past answers have not been well-received, and you're in danger of being blocked from answering.


            Please pay close attention to the following guidance:


            • Please be sure to answer the question. Provide details and share your research!

            But avoid



            • Asking for help, clarification, or responding to other answers.

            • Making statements based on opinion; back them up with references or personal experience.


            To learn more, see our tips on writing great answers.




            draft saved


            draft discarded














            StackExchange.ready(
            function () {
            StackExchange.openid.initPostLogin('.new-post-login', 'https%3a%2f%2fmath.stackexchange.com%2fquestions%2f179997%2fpythagorean-triplets-with-bounds%23new-answer', 'question_page');
            }
            );

            Post as a guest















            Required, but never shown





















































            Required, but never shown














            Required, but never shown












            Required, but never shown







            Required, but never shown

































            Required, but never shown














            Required, but never shown












            Required, but never shown







            Required, but never shown







            Popular posts from this blog

            Biblatex bibliography style without URLs when DOI exists (in Overleaf with Zotero bibliography)

            ComboBox Display Member on multiple fields

            Is it possible to collect Nectar points via Trainline?